L'intégrale de Kurzweil-Henstock

13»

Réponses

  • Bonjour.

    Je profite de la présence des balaises de l'intégration pour poser ma question.

    J'ai une fonction $f$ dérivable à droite sur un intervalle $[a,b[$. Est-ce que $f^\prime_d$ est KH-intégrable sur $[a,c]$ avec $a<c<b$ ?

    e.v.
    Personne n'a raison contre un enfant qui pleure.


  • Je relance.

    Peut-on démontrer de façon simple/élémentaire, que si \( g \) est une fonction convexe sur \( ]a,b[ \),
    \[ \forall (x,y) \in\, ]a,b[^2, \qquad \int_x^y g^\prime_d(t) \, \mathrm dt = \int_x^y g^\prime_g(t) \, \mathrm dt = g(y) - g(x) \quad?

    \] Il va de soi que \( g^\prime_d \) et \( g^\prime_g \) sont croissantes donc intégrables et qu'elles diffèrent sur un ensemble au plus dénombrable, ce qui assure l'égalité des deux intégrales.

    Que le paraclet soit profitable aux personnes concernées.
    e.v.

    [ Chez moi il apporte la flotte. ]
    Personne n'a raison contre un enfant qui pleure.


  • Bonjour ev, il semble que tu veux rédiger un document sur KH avec pleins de propriétés, c'est ça ?
    Le 😄 Farceur


  • Bonjour Gebrane.

    Je cherche des exemples où K et H font mieux de Riemann. (Normal, à deux contre un).
    Le mieux : des exemples comme le premier que j'ai proposé (bon, le tuyau était un pneu crevé) où l'intégrale n'apparait pas dans l'énoncé.

    Amicalement,

    e.v.
    Personne n'a raison contre un enfant qui pleure.


  • Têtu, moi ?
    Dans l'excellent "Une année de colles en Math Sup MPSI" de l'excellent Eric on trouve l'exercice 23.7 page 575 (24.9 page 577 de la première édition).

    Soit \( f \in \mathcal C_{[0,1]}^1 \). Montrer que
    \[ \lim _{n \rightarrow+\infty} n\Big(\frac{1}{n} \sum_{i=1}^{n} f\big(\frac{i}{n}\big)-\int_{0}^{1} f(x) d x\Big)=\frac{f(1)-f(0)}{2}.

    \] Peut-on affaiblir l'hypothèse avec \( f \) dérivable sur \( [0,1] \) ?

    Amicalement,
    e.v.

    [ C'est pas d'la pub, ça ? (...) Un lien, tu veux un lien ??? ]
    Personne n'a raison contre un enfant qui pleure.


  • Je te remercie e.v. pour le compliment.

    Comme le remarque Masayoshi Hata (Problems and solutions in real analysis, 1st ed., ex 5.7), on ne peut pas se contenter de la seule continuité (la fonction de Takagi donne un contre-exemple). De plus, la solution qu'il donne de l'exercice utilise l'uniforme continuité de $f'$ sur $[0,1]$, d'où l'hypothèse $C^1$ sur $f$. Peut-on ne pas utiliser l'uniforme continuité de $f'$ ?
  • Bonsoir Eric.
    Ah, ça y est, je l'ai !
    Avec tes notations :
    \[ m_{i} :=\inf \left\{f^{\prime}(x): x \in\left[\frac{i-1}{n}, \frac{i}{n}\right]\right\}
    \] et
    \[ M_{i} :=\sup \left\{f^{\prime}(x): x \in\left[\frac{i-1}{n}, \frac{i}{n}\right]\right\}.
    \] La fonction \( f^{\prime} \) est KH-intégrable sur \( [0,1] \), donc les
    \[ \frac1{2n}\sum_{i=1}^{n} m_{i} \; \text{ et } \; \frac1{2n}\sum_{i=1}^{n} M_{i}
    \] sont bien des sommes de Riemann qui convergent vers \( \displaystyle \frac{1}{2} \int_{0}^{1} f^{\prime}(x) d x
    =\frac{f(1)-f(0)}{2} \), ...

    Sauf qu'au départ, l'existence des \( m_{i} \) et \( M_{i} \) n'est pas assurée pour une fonction banalement dérivable.

    Encorre rraté.
    Bonne nuit,
    Général Alcazar.
    Personne n'a raison contre un enfant qui pleure.


  • ev tu peux supposer de plus que f' est borné
    Le 😄 Farceur


  • Bon, il est temps de faire chauffer la babasse.

    Je considère une fonction \( g \) définie sur \( ]0,1] \) par \( g(x) = x^7\sin\left( \dfrac\pi{x^4} \right) \) et \( g(0) = 0 \).
    La fonction \( g \) est dérivable sur \( [0,1] \) et
    \( f(x) := g'(x) = 7x^6\sin\left( \dfrac\pi{x^4} \right) - 4\pi x^2 \cos\left( \dfrac\pi{x^4} \right) \).
    On a donc \( f \) dérivable sur \( [0,1] \) mais \( f' \) n'est pas continue en zéro.
    De plus \( \displaystyle\int_0^1 f(t) \, \mathrm dt = \int_0^1 g'(t) \, \mathrm dt = g(1) - g(0) = 0 \).
    Montrer que
    \[ \lim_{n \rightarrow+\infty} n\Big(\frac{1}{n} \sum_{i=1}^{n} f\big(\frac{i}{n}\big)-\int_{0}^{1} f(x) d x\Big)=\frac{f(1)-f(0)}{2} \] c'est montrer que
    \[ \lim_{n \rightarrow+\infty} \sum_{i=1}^{n} f\big(\frac{i}{n}\big) =\frac{f(1)-f(0)}{2} = 2\pi. \]
    Je programme le calcul de la somme \( \displaystyle\sum_{k=1}^{2^n} f\big(\frac{k}{2^n}\big) \) :
    1 9.42477796076938
    2 14.226450595563698
    3 7.350862740000039
    4 6.019044902834182
    5 1.8439427218585056
    6 2.810064602302168
    7 9.932561699721935
    8 14.554581778577052
    9 8.900509033264624
    10 5.283809207430888
    11 7.242564057308822
    12 13.335558623246783
    13 1.0669089862686523
    14 -1.1034015167778097
    15 9.709071639906298
    16 -3.7289133813154667
    17 -7.8552827799027884
    18 -4.487584324587491
    19 5.283524333171092
    20 8.388453305749046
    21 -4.110394428691853
    22 -6.473644277517529
    23 -9.739125996295423
    24 -9.338914315543567
    25 -7.381176851526913
    
    On voit que ce n'est pas un algorithme performant pour calculer \( 2\pi \).

    Amicalement,

    e.v.
    Personne n'a raison contre un enfant qui pleure.


  • La question se pose de savoir si la suite est erratique ou si c'est le programme qui n'est pas assez robuste.

    Le même programme avec \(\quad g(x) = x^5\sin\left( \dfrac\pi{x} \right) \) et \( g(0) = 0 \).
    La fonction \( g \) est dérivable sur \( [0,1] \) et
    \( f(x) := g'(x) = 5x^4\sin\left( \dfrac\pi{x} \right) - \pi x^3 \cos\left( \dfrac\pi{x} \right) \).
    On a donc \( f \) dérivable sur \( [0,1] \) et \( f' \) est continue en zéro cette fois.
    On est dans les clous de l'exercice d'Eric.

    On a toujours \(\quad \displaystyle\int_0^1 f(t) \, \mathrm dt = \int_0^1 g'(t) \, \mathrm dt = g(1) - g(0) = 0 \).
    Montrer que \[ \lim_{n \rightarrow+\infty} n\Big(\frac{1}{n} \sum_{i=1}^{n} f\big(\frac{i}{n}\big)-\int_{0}^{1}
    f(x) d x\Big)=\frac{f(1)-f(0)}{2} \] c'est montrer que
    \[ \lim_{n \rightarrow+\infty} \sum_{i=1}^{n} f\big(\frac{i}{n}\big) =\frac{f(1)-f(0)}{2} = \frac \pi2.

    \] Le programme de calcul de la somme \(\quad \displaystyle\sum_{k=1}^{2^n} f\big(\frac{k}{2^n}\big) \) donne :
    1 2.7488935718910694
    2 1.992406634964477
    3 1.8166491899836257
    4 1.684491140957376
    5 1.633187854071759
    6 1.6066405167840734
    7 1.586102159137793
    8 1.5788455952321492
    9 1.574775892133999
    10 1.572923205725059
    11 1.5717870189388443
    12 1.5713164468044547
    13 1.5710445270814772
    14 1.5709214943285335
    15 1.5708591340540443
    16 1.5708278257050523
    17 1.5708120660119538
    18 1.5708042688790544
    19 1.5708002900960398
    20 1.5707983276866053
    21 1.5707973241639022
    22 1.5707968388080995
    23 1.5707965811938833
    24 1.570796562029134
    25 1.5707966063577015
    

    Ce n'est pas flamboyant comme convergence, mais on retrouve \( \frac \pi2 \).
    Amicalement,

    e.v.
    Personne n'a raison contre un enfant qui pleure.


  • ev
    ev
    Modifié (January 2022)
    Un exercice (classique ?)
    Soit \( f \) une fonction dérivable de \( \R \) dans \( \R \).
    On suppose que \( \lim\limits_{x\to+\infty} (f'(x)+f(x)) = \ell \in \R \).
    Démontrer que \( \lim\limits_{x\to+\infty} f(x) = \ell \).
    Amicalement,
    e.v.
    Personne n'a raison contre un enfant qui pleure.


  • Bonjour @ev, ton exercice est un classique de colle et permet de voir le danger de la règle de l’Hôpital
    Le 😄 Farceur


  • L'exercice est bien classique (il est récurrent en taupe sauf erreur). Pour les inégalités (lorsque on a une fonction dérivable, même si l'intégration de Lebesgue ne s'applique plus), l'intégrale de HK n'apporte pas plus que l'inégalité des accroissements finis (même si elle guide les calculs). La difficulté de cet exercice est surtout de penser à la bonne fonction auxiliaire.
    Une fonction est un ensemble $f$ de couples tel que pour tous $x,y,z$, si $(x,y)\in f$ et $(x,z)\in f$ alors $y = z$.
  • @Foys l'exercice se fait avec les epsilons
    Le 😄 Farceur


  • Foys
    Modifié (January 2022)
    Je passe par l'édude de la fonction $x\mapsto e^{-x} g'(x)$ où $g(x)=e^xf(x)$ pour tout $x\in \R$. Je me demande s'il est possible d'éviter cette astuce.
    Une fonction est un ensemble $f$ de couples tel que pour tous $x,y,z$, si $(x,y)\in f$ et $(x,z)\in f$ alors $y = z$.
  • gebrane
    Modifié (January 2022)
    On écrit la définition que f+f' tend vers l, on multiplie les deux inégalités par l'expo, on intègre et voilà.
    Le 😄 Farceur


  • Chaurien
    Modifié (January 2022)
    Attention Gebrane, pour intégrer il faut que la dérivée soit intégrable, disons que la fonction soit $\mathcal C^1$.
  • gebrane
    Modifié (January 2022)
    @Chaurien ah oui c pas C^1
    Le 😄 Farceur


  • Tout dépend du type d'intégrale. Pour KH, toutes les dérivées sont intégrables, mais ce n'est pas (encore ?) dans les programmes d'enseignement.
  • @ev je viens de vérifier dans mes archives. On suppose que f est C^1; donc ce n'est pas un classique de colle,  je suis désolé pour ma réponse hâtive. Je suis vraiment nul de répondre sans faire attention aux hypothèses.
    Le 😄 Farceur


  • ev
    ev
    Modifié (January 2022)
    Je vais essayer de montrer que la fonction auxiliaire apparaît naturellement.
    On commence par supposer que \( \ell = 0 \). On pose \( \varepsilon(x) := f'(x)+f(x) \).
    La fonction \( f \) est solution de l'équation différentielle
    \[ y' + y = \varepsilon(x). \]
    On résout l'équation homogène : \( y(x) = k \mathrm e^{-x} \).
    On résout l'équation par la méthode de la variation de la constante :
    \[ \begin{array}{lrcl}
    &y'(x) &=& k'(x)\mathrm e^{-x} - k(x)\mathrm e^{-x}\\
    \text{donc} & k'(x)\mathrm e^{-x}&=& \varepsilon(x)\\
    \text{soit} & k'(x)&=& \varepsilon(x)\mathrm e^{x}.
    \end{array} \]
    Or on a \[ \forall x \in \R, \; \varepsilon(x)\mathrm e^{x} = \left( f'(x)+f(x) \right)\mathrm e^{x} = \dfrac{d}{dx} \left( f(x) \mathrm e^{x}\right). \]
    La fonction \( k' : x \longmapsto \varepsilon(x)\mathrm e^{x} \) est une dérivée, elle est donc intégrable (KH) sur tout segment \( [x_0~,~x] \) et on a
    \[ k(x) - k(x_0) = \int_{x_0}^x \varepsilon(t)\mathrm e^{t} \, \mathrm dt. \]
    Il est temps de se souvenir que \( \lim\limits_{x\to+\infty} \varepsilon(x) = 0 \).
    Soit \( \varepsilon > 0 \). Il existe \( A \in \R \) tel que \( \forall x \geqslant A \), on a \( \left \vert \varepsilon(x) \right\vert \leqslant \varepsilon \). On en déduit qu'en prenant \( x \geqslant A \),
    \[ \left \vert k(x) - k(A) \right\vert \leqslant \int_{x_0}^x \mathrm e^t \varepsilon  \, \mathrm dt \leqslant \varepsilon \left( \mathrm e^x - \mathrm e^A \right) \leqslant\varepsilon \mathrm e^x. \]
    On en déduit que \( \lim\limits_{x\to+\infty} \left( k(x) - k(A) \right)\mathrm e^{-x} = 0 \).
    Comme \( \lim\limits_{x\to+\infty} k(A)\mathrm e^{-x} = 0 \), on en déduit que \( \lim\limits_{x\to+\infty} k(x)\mathrm e^{-x} = 0 \), ce qui permet de conclure puisque \( k(x)\mathrm e^{-x} = f(x) \).
    Dans le cas général, on considère la fonction \( \tilde f := f - \ell \) et on applique la méthode du jaguar casqué.
    Amicalement,
    e.v.
    Personne n'a raison contre un enfant qui pleure.


  • Rendons la tache ardue
    Soit \( f \) une fonction n fois dérivable de \( \R \) dans \( \R \).
    On suppose que \( \lim\limits_{x\to+\infty} (f(x)+f'(x)+...+f^{(n)}(x)) = \ell \in \R \).
    Démontrer que \( \lim\limits_{x\to+\infty} f(x) = \ell \).
    Amicalement,
    geb
    Foys, je pense que tu prends $g(x)=e^x ( f(x)-l)$,

    Le 😄 Farceur


  • gebrane
    Modifié (January 2022)
    @ev On peut simplifier ta méthode. Soit $f+f'=g$, puisque que $g$ est KH intégrable, on peut appliquer la méthode de la variation des constantes qui dit que $$\forall x\in\R,\quad f(x)=f(0)e^{-x}+e^{-x}\int_0^x e^t g(t) dt,$$
    $ g-l$ tend vers $0$ en $+ \infty$, donc $\forall \epsilon >0$, il existe $A>0$, $\forall x\geq A,\ |g(x)-l|<\epsilon$. On veut démontrer que $$\lim_{x\to +\infty}e^{-x}\int_0^x e^t g(t) dt=l,$$ on a $\displaystyle e^{-x}\int_0^x e^t g(t) dt= e^{-x}\int_0^x e^t (g(t)-l) dt  +l(1-e^{-x})=e^{-x}\int_A^x e^t (g(t)-l) dt  +e^{-x}\int_0^A e^t (g(t)-l) dt+l(1-e^{-x}).$  Il suffit de démontrer que $$\lim_{x\to +\infty}e^{-x}\int_A^x e^t (g(t)-l) dt=0.$$
    $\forall x>A$ on a $|e^{-x}\int_A^x e^t (g(t)-l) dt|\leq e^{-x}\int_A^x e^t \epsilon dt=(1-e^{-(x-A)})\epsilon$, donc $$\limsup_{x\to+\infty} |e^{-x}\int_A^x e^t (g(t)-l) dt| \leq \epsilon.\quad CQFD$$
    J'ai une preuve utilisant la deuxième formule des accroissements finis, si cela intéresse quelqu'un,  je peux la rédiger pour voir si c'est juste.
    Le 😄 Farceur


  • gebrane
    Modifié (January 2022)
    Dans mon raisonnement j'ai utilisé sans le prouver que si g est KH intégrable alors $e^x g$ est aussi KH intégrable. peut être j'ai commis une bêtise car je me rappelle que @Math Coss  avait donné un contre de la non intégrabilité KH du produit $hg$ si $h$ est continue et $g$ KH.
    Le 😄 Farceur


  • ev
    ev
    Modifié (January 2022)
    Bonjour Gebrane.
    Je ne suis pas convaincu que ta méthode soit plus simple, ni même qu'elle soit différente...
    Tu as l'intégrabilité de $e^xg$ dans la mesure où $g$ est minorée -- peut-être pas à partir de zéro, mais au moins à partir de ton $A$.
    Amicalement,
    e.v.
    Personne n'a raison contre un enfant qui pleure.


  • gebrane
    Modifié (January 2022)
    @ev vois-tu comment appliquer la méthode de foys avec la fonction auxiliaire expo. J'ai une preuve avec AF.
    Le 😄 Farceur


  • ev
    ev
    Modifié (January 2022)
    Bonjour Gebrane.
    Pour ton exercice je propose le plan suivant :
    1/ On démontre par récurrence sur \( n \in \N \) la proposition \( \mathcal P_n \) : pour toute fonction \( f \) , \( \; n \) fois dérivable sur \( R \), pour tout \( a_1, \ldots, a_n) \in \R_+^* \),  \( \; \lim\limits_{x\to+\infty} \left( f(x) + a_1f'(x) + \ldots + a_nf^{(n)}(x) \right) = \ell \Longrightarrow \lim\limits_{x\to+\infty} f(x) = \ell \).
    2/ Pour démontrer \( \mathcal P_n \Longrightarrow \mathcal P_{n+1} \), on commence par étudier le cas des suites \( 1 < a_1 < \ldots < a_{n+1} \).
    Amicalement,
    e.v.
    Personne n'a raison contre un enfant qui pleure.


  • gebrane
    Modifié (January 2022)
    Bon, je dois revoir mon propre exercice à la baisse car c'est faux pour n>3 ( Prendre n=4 et f(x)=cos(x))
    Je me contente donc de l'exercice suivant et je crois capable le traiter
    Soit \( f \) une fonction 2 fois dérivable de \( \R \) dans \( \R \).
    On suppose que \( \lim\limits_{x\to+\infty} (f(x)+f'(x)+f''(x)) = \ell \in \R \).
    Démontrer que \( \lim\limits_{x\to+\infty} f(x) = \ell \).
    Pourquoi je commets tant de bêtises surtout dans ce  fil de @ev, mystère ! 
    Le 😄 Farceur


  • Deux fois dérivable, ou bien de classe $C^2$ ?
  • Bah @Chaurien c'est comme dans le cas d'une fonction dérivable et non pas C^1, @ev veut nous montrer les secrets du KH
    Le 😄 Farceur


Connectez-vous ou Inscrivez-vous pour répondre.